« first day (1505 days earlier)      last day (3513 days later) » 

12:09 AM
Quick question guys. For a function $y=f(x)$ that is rotated about the $X$-Axis from $x=a$ to $x=b$, is the surface area not given by $$ \mathrm{SA} = 2\pi \int_{a}^{b} f(x) dx ?$$
 
@GustavoMontano I don't think so.
 
I just drew a diagram and that is what I am getting...unless I am making a little assumption.
 
Oh, wait.
 
Ah yes! I was making a poor assumption there.
Makes sense :).
No I think you're right.
Well..
 
You said surface area yes?
 
Right, you're missing the arc-element.
 
Mathworld gives that formula. They look equivalent except for the part where they have $ds$ instead of $dx$
That that's it.
 
Yes, $ds$ is the arc-element.
Wanna try again?
 
And that makes sense. :)
Will do, thanks! I assumed since $dx$ is small enough it will be a straight cylinder.
Bad Gustavo
 
OK, let me know if you need help,
 
12:12 AM
Thanks Pedro ^_^
 
@Hippalectryon @sarah Stop filling the chat with nonsense. It's like having a party in a public park and then leaving trash all over the place.
5
 
12:49 AM
Nonsense
 
r9m
1:15 AM
@robjohn :-) Nice !!
 
How would you describe "negatively curved" part of a 3d surface.
Is there a mathematical unit I can attach to this?
 
@studentmath it's this question math.stackexchange.com/questions/931921/…
 
Oh, curvature of course....
 
2:01 AM
Hi @Alex @Pedro
 
2:41 AM
hey guys, how does (2[latex]\sqrt 5i[/latex])(-2[latex]\sqrt 5i[/latex]) yield 20? wouldn't you get 4[latex]\sqrt 25i^{2}[/latex] = [latex] 4 \times 5 \sqrt i^{2} [\latex]= [latex]4 \times 5 \times i [\latex] ?
sorry about the latex fail.... there's no preview button....
\sqrt i^{2}
..... phail.
 
@WalrustheCat $\sqrt{5}^2 = 5$ and $-2(2)i^2$ = 4
 
i guess i'm asking how you're getting the i squared
i can see sqrt(i squared)
which i'd think would yield i?
no wait i see it
sorry thanks
 
No problem.
 
2:57 AM
yo guys
how do I plot complex variables in a polar graph?
 
no, wait, i don't. if $\sqrt(5) \times \sqrt(5) = 5 $, wouldn't $\sqrt(i) \times \sqrt(i) = i $? so $ 2 \times -2 \times 5 \times i = -20i $ ?
 
Your formatting is confusing me. Your above latex had $\sqrt 5i$, so i wasn't under the root
 
so basically
easy stuff
now to my question
 
i'll rewrite the question now that i have a handle on the latex thanks
i have in front of me that $( 2 \sqrt(5i)) \times ( -2 \sqrt(5i)) = +20 $ . i don't see it. i expand that to $ 2 \times -2 \times \sqrt(25 \times i^{2}) = -4 \times 5 \times \sqrt(i^{2}) = -20i $ ? where am i going wrong? thanks!
dude chantry -- you are so right. i'm reading the problem wrong. the i is not under the root. thx again
w00sh!
 
wow
much woosh
so zeros
 
3:09 AM
sw00sh.
 
alright now
if I have 5-10i
how do I graph that in the polar coordinate system?
Same as orthogonal?
 
I feel like that's just normal complex graphing, isn't it?
 
That's what I feel
but that's not a good feeling at all
 
Maybe they want you to calculate the angle and whatnot
 
err, I'll take your word for it being the same
 
3:13 AM
Hey, don't put it on me.
 
 
2 hours later…
4:45 AM
Gosh, I am having hard time understanding this question:
For a surface $S$ in Monge form $z=f(x,y)$, find a formula for the area of the piece of surface that lies over a region $D$ in the $XY$ plane.
Can anyone give me their interpretation?
I mean, judging from what has been given, I can only assume it is asking me to generalise a formula for surface area of a surface through projections.
That is, projecting a rectangular piece of area on the normal XY plane to the 3D surface.
 
seems you understand the question
you project a region D in the xy plane up/down to the surface
and ask for its surface area
 
Great! Thanks @anon.
I have to admit, this proof is amazing.
I did find it somewhere, and it is beautifully derived.
 
 
1 hour later…
5:50 AM
does anyone know of any good books that would help me transition from undergrad stat to grad lvl stat...
 
@user52932 i haven't read it but "All of Statistics" is supposed to be good
 
6:48 AM
Mystical greetings
 
is there anything wrong with the following argument? if $g(n)=f(n-1)-f(n)$, then $$\sum_{n=1}^\infty n g(n)=\sum_{n=1}^\infty n\left[f(n-1)-f(n)\right]=\sum_{n=1}^\infty \left[(n+1)f(n)-nf(n)\right]=\sum_{n=1}^\infty f(n)$$
i know i'm probably supposed to make it a limit of partial sums or something but i really can't see how it could possibly go wrong
 
What?
Does that even make sense intuitively?
 
@GustavoMontano yeah
look at the 1st term in the summand in the 2nd inequality
$nf(n-1)$ is $(n+1)f(n)$ if I move it to the previous $n$
so i just move 'em all down
 
 
2 hours later…
9:06 AM
Hi everybody! :-)
 
9:46 AM
Hi! :-)
 
0
Q: A class of function to study Fourier analysis, which is a subset of BV functions.

Rajesh DIn Fourier analysis, while talking about pointwise convergence, we generally start with the class of functions called, BV functions (functions of bounded variation), which have a finite total variation. I'd like to avoid this type of classification as they are more complex and I don't really nee...

 
10:08 AM
@Chris'ssis I think you misunderstood my question. I'm looking for series representation of $\dfrac{\ln(1-x)}{1+x}$ that involving harmonic number, not its integral
 
10:24 AM
like differentiating both sides of $$\int_0^x \dfrac{\ln(1-t)}{1+t}\ dt =\sum_{k=1}^{\infty} \sum_{n=1}^{\infty} (-1)^{n+1}x^{k+n} H_k \left(\frac{x}{k+n+1}-\frac{1}{k+n}\right)$$
?
@robjohn this morning I derived some results absolutely crazy
 
11:20 AM
@Anastasiya-Romanova $$\dfrac{\ln(1-x)}{1+x} =\sum_{k=1}^{\infty} \sum_{n=1}^{\infty} (-1)^{n+1} (x-1) x^{n+k-1} H_k$$
@Anastasiya-Romanova and $$\dfrac{\ln(1+x)}{1-x} =\sum_{k=1}^{\infty} \sum_{n=1}^{\infty} (-1)^{k+1} (x+1) x^{n+k-1} H_k$$
 
11:35 AM
@Balarka Alright :|
 
12:02 PM
Of couse, if you want you can write them as a single series like $$\dfrac{\ln(1-x)}{1+x}=\sum_{n=1}^{\infty} \frac{(x-1)x^n H_n}{1+x}$$
 
@Chris'ssis Could you please elaborate a bit more for the latter term?
 
@Anastasiya-Romanova see here $$\dfrac{\ln(1+x)}{1-x}=\sum_{n=1}^{\infty} \frac{(-1)^{n+1} (1+x)x^n H_n}{1-x}$$
 
Here?
I still don't get it.
 
@Anastasiya-Romanova All you need to know is the simple fact that $$\sum_{n=1}^{\infty} x^n H_n =-\frac{\log(1-x)}{1-x}$$
 
@Sawarnik Long story
 
12:11 PM
Hmm.
 
@Chris'ssis Hhmmm. Let me think first
 
@Anastasiya-Romanova by the way, I'd like to see if Cleo is able to do this one without using special functions $$\int_0^1 \frac{\log(1-x) \log(1+x)}{1+x} \ dx$$ If you ask me, I think Cleo is not able to do it like that.
brb
 
@Chris'ssis How if I ask that as a new OP? Right now, I'm making a new one. Will be posted soon.
 
12:54 PM
Sorry to interrupt, But could you guys please have a look at my problem Locus of Orthocenter?
 
@DanielFischer Hi Sir... (ô‿ô)
 
Hi @Anastasiya-Romanova.
 
@DanielFischer You often stop in here but you barely chat with us, so I decide to greet you
You have lots of rep both on Stack Overflow & MSE. Cool!
 
1:10 PM
I'm not good at small talk, so I mostly only say something if I have something to say. Or want to post a bad pun.
Answering a lot of questions tends to have that effect ;)
 
@Anastasiya-Romanova I computed that one with the high school tools only (of course, I used the definition of $\zeta(3)$, but this wouldn't be a problem).
 
@DanielFischer Speak less, do more. You kind of a cool guy. Like Sasuke!
 
Who is Sasuke?
 
@Chris'ssis Let me try first before I post it
 
@Anastasiya-Romanova Sure, take your time. Try it as long as you want.
 
1:45 PM
@Chris'ssis $$\frac{\ln^32}{3}-\frac{\pi^2}{12}+\frac{\zeta(3)}{8}$$
 
@Anastasiya-Romanova Not really ...
 
@Chris'ssis Are you sure?
 
@Anastasiya-Romanova $$\frac{\ln^32}{3}-\frac{\pi^2}{12}\log(2)+\frac{\zeta(3)}{8}$$ But as I told you, computing this integral by special functions is not interesting to me. So, knowing the answer is not enough. :-)
 
@Chris'ssis Sorry, I miss the factor $\ln2$
 
@Anastasiya-Romanova Did you do it without special functions?
 
1:52 PM
@Chris'ssis I use school method
With magic substitution.
 
@Anastasiya-Romanova What would be that one?
 
By part and series expansion
Is it okay if I use series expansion?
 
@Anastasiya-Romanova I didn't use it.
 
Seriously? How come then you get the factor $\frac{\pi^2}{12}$ & $\zeta(3)$ if not using series expansion?
 
@Anastasiya-Romanova By using some results from my personal research (at the high school level).
 
1:59 PM
Sorry, I doubt it.
 
@Anastasiya-Romanova Never doubt it.
 
Then how you arrive to that answer without using series expansion? Double integral?
 
@Anastasiya-Romanova I use some double integral (to prove a result that I use in the main proof).
 
After substitution I get this integral $$\int_0^1\frac{\ln(1-x)}{x}dx$$. We can use series expansion to evaluate this one but if you use double integral, then maybe you use this method: math.stackexchange.com/a/891300/133248
The problem is that method is using multivariable calculus, we also need to use Jacobian
I also get this integral $$\int_0^1\frac{\ln(1-x)\ln x}{x}dx$$
I wonder, what kind of double integral did you use to evaluate those two without using Jacobian
What kind of high school in this world taught its student about Jacobian?
 
2:14 PM
No need for the Jacobian you learn about in the multivariable calculus.
 
@Anastasiya-Romanova May I give you a hint? Only if you want ...
 
Yes, please
 
That particular blog of Baez always fascinated me.
 
@Anastasiya-Romanova Make you of the following relation that comes from my research
 
2:17 PM
@BalarkaSen Is that plot of all the roots of all polynomials of degree ⩽5 with integer coefficients ranging from −4 to 4?
 
@Anastasiya-Romanova see above
 
@Anastasiya-Romanova Yes, did you see the blogpost too?
 
@Chris'ssis That series expansion
@BalarkaSen What blog? I saw that plot somewhere on MSE
 
Fair enough, have a look
 
@BalarkaSen The plots are really beautiful
 
2:20 PM
I don't care about beauty. The fact that they resembles well-known fractals are much cooler.
The holes at rationals are consequence of diophantine errors : one of the characteristics of algebraic numbers are that they can't be arbitrarily well approximated by rationals.
 
@BalarkaSen I do care about beauty since I am a girl
 
@Anastasiya-Romanova Also my questions are about beauty (like the last result). :-)
 
I have never looked at the math they did there, but to my understanding appearance of fractals aren't unusual. The fractal-like formations are due to polynomial iterations from an accumulation point.
@Anastasiya-Romanova I am just interested on the math.
 
@Chris'ssis You use series expansion & double integral. Indeed, they're beauty.
@BalarkaSen Are you saying that you're not interested in beautiful girls?
 
@Anastasiya-Romanova There was a misunderstanding above, I thought you were referring to $\log(1\pm x)$.
bbl (a break is needed)
 
2:30 PM
@Chris'ssis What kind of misunderstanding?
I only ask how come you get the closed form of your integral without using series? You answer without using it, you said you used double integral.
 
@Anastasiya-Romanova I thought you were referring of using the series of $\log(1\pm x)$ I didn't use.
 
So how you evaluate it then? Could you please elaborate?
 
@Anastasiya-Romanova Sorry, I cannot tell you more now. You'll find that in my book that I'll publish somewhere during the next year if I work very hard.
 
@Anastasiya-Romanova nah, I much like mathematics.
 
@DanielFischer You didn't see my mistake now. It is "you were referring to ". ;)
 
2:39 PM
@Chris'ssis I have an excuse: I wasn't on the chat tab.
 
lol :-)
 
@Chris'ssis But I only asked one problem, not the whole of your book. You have got nothing to lose
 
@Anastasiya-Romanova The key is to cleverly use the relation I gave you above.
 
But the relation you've shown to me is using series expansion. You said you didn't use it
 
@DanielFischer abc-conjecture tells that there are no more than $2$ perfect powers appearing in $(n^2, (n+1)^2)$ for all positive integer $n$.
in fact, there seems to be no more than $3/2\sqrt{n} + O(1)$ perfect powers in $(n^3, (n+1)^3)$ (conditional on abc)
 
2:44 PM
Aha, @BalarkaSen. Is it easy to derive that from the ABC conjecture?
 
More or less.
 
@Anastasiya-Romanova I gave you some clarifications above. I initially thought you were only referring to the series of $\log(1\pm x)$. I didn't use the series of $\log(1\pm x)$, that is my point.
 
@Chris'ssis: very nice
 
@DanielFischer let $a, b$ be perfect powers $n^2 < a < b < (n+1)^2$. as cubes grow much faster than sqaures, it is expected that for large $n$ there are no more than a singe cube inside teh interval. WLOG, $a$ is a cube and $b$ is a fifth power. then $rad(a) = O(n^{2/3})$ and $rad(b) = O(n^{2/5})$. Let $c = b - a$ which is $< (n+1)^2 - n^2 = O(n)$. Thus $rad(abc)^{1+\epsilon} = O(n^{2/3} \cdot n^{2/5} \cdot n)^{1+\epsilon} = O(n^{31/30+\epsilon})$.
 
@robjohn :D Those results I got this morning.
 
2:47 PM
But then $b > n^2$, hence there are finitely many such triples conditionally on abc
Hence there are no more than $2$ perfect powers in $(n^2, (n+1)^2)$ with at most finitely many exceptions
 
@Anastasiya-Romanova you can use all series expansions you want, no problem. Just let me know if you get that result without using special functions.
 
I didn't use any special functions
Substitution, series expansion & by parts
 
@Chris'ssis Do these results have any unifying concepts, or are they just done piecemeal?
 
@robjohn They are based upon something I study these days, at least the last $2$ results. Using that tool, I derived both series.
 
hello I have a really quick question about Poisson can i hit it?
anyone?
I hope someone eventually will see it: We have 3 mails arriving per hour (with a poisson distribution), 2 hours pass by without receiving mails, what is the expected time that we'll get the 1st mail? My answer is that be cause it is memoryless it will wait constantly 1/3 of the hour ---> 20 minutes, a friend says since it's memoryless and 2 hours pass by that we can't find the time. is any oh these answers correct?
 
3:07 PM
@user2692669 Your intuition about the lack of memory is correct (because you are talking about the expected wait)
 
I just watched "If I live". It is the best movie I ever watched.
 
Huy
Good afternoon, everyone.
 
@WillHunting never seen it.
 
I see that many pictures got deleted from this chat, lol.
 
@Huy Huy
 
3:09 PM
@Huy good morning :-)
 
@WillHunting Yes, since you guys were spamming
 
@WillHunting by owner, or moderator?
 
@BalarkaSen Well, the pics were harmless.
@robjohn Probably by mod. We went a bit wild.
 
@Will Apparently they were nonsense. make up a nonsense room and post them there.
 
@BalarkaSen Look, most of this chat is nonsense.
 
3:10 PM
it was deleted by a mod. by Alexander, in fact
 
@WillHunting right on topic for fall break in South America ;-)
 
15 hours ago, by Alexander Gruber
@Hippalectryon @sarah Stop filling the chat with nonsense. It's like having a party in a public park and then leaving trash all over the place.
 
@robjohn so... it's 20 minutes or...?
 
@user2692669 if the distribution is Poisson with 3 events per hour, then yes.
 
I am so inspired by the movie. Time to change my life, lol.
I will continue to look out for my enemy's nonsense comments, but I suspect a mod told her to stop posting them yesterday after I flagged 10 of them.
I was hoping she would get suspended, but a suspension would be too much for such a small thing, lol.
 
r9m
3:12 PM
@DanielFischer I see you had been silenced ... ;) =)
 
Why are you referring to a##hy as "her", @Will?
 
@ro
 
@BalarkaSen I don't wanna be so obvious.
 
@robjohn yes it is, and we don't care about how much time passes without getting a message,right? even if 10^321 hours pass by the system will be waiting about 20 minutes (?)
 
@user2692669 If the distribution is as described, the expected wait should not change based upon history. However, one may question the validity of the model if $10^{321}$ hours pass without a single event.
 
3:18 PM
hehehe yes, thank you :)
 
r9m
@robjohn can you check if I made any mistakes here ?! :) also I read the nice answer you linked to me today .. I am taking an introductory ANT course this sem .. we are learning to use Euler-Maclaurin summation ! :)
 
@Chris'ssis Hello
 
@Hippalectryon Hi
bbl (I'm in a meeting now)
 
@r9m The EMSF is a wonderfully useful tool :-)
 
r9m
@robjohn ya ! we are learning how to use it to estimate various series :) really useful tool !
 
3:27 PM
@Chris'ssis For when you're back :Let $f$ be a continuous, decreasing function, with a $0$ limit in $\infty$. Let $\displaystyle x>0,\gamma_k(x)=xf((k+1)x)-\int_{(k+1)x}^{(k+2)x}f(t)\mathrm{d}t$. Let $\Gamma(x)=\sum\limits_{k=0}^{\infty}\gamma_k(x),\lim_{x\rightarrow0} xf(x)=A$. What is $\displaystyle\lim_{x\rightarrow\infty}\Gamma(x)$ ?
@r9m emsf.ae ?
 
r9m
@robjohn could you recommend me a book where I could start reading Fourier Analysis ? :)
 
@r9m You should study real analysis first.
 
yuck real analysis
 
I am trying to like every major branch of math.
 
@r9m Katznelson's An Introduction to Harmonic Analysis is a good book
 
r9m
3:32 PM
@WillHunting okay .. I had an introductory course in first sem and another one in 3rd sem .. but I wan't to learn a bit about fourier analysis for my ANT classes :-)
@robjohn o!! Thank you very much !! :D
 
@r9m You can try Stein's Fourier Analysis in the Princeton Lectures in Analysis series.
 
You have class with ants ? :)
 
@r9m analytic number theory?
 
r9m
@BalarkaSen ya
 
excellent.
 
3:33 PM
Stein has 4 books in that series.
 
i learned fourier analysis from Titchmarsh.
 
r9m
okay ! :)
 
@WillHunting And four series in that book ?
:D
 
Spare us!
 
@Hippalectryon Chris's Sis's book? =)
 
3:34 PM
Hehe
 
r9m
@WillHunting thanks a lot !! :D
 
BTW, @r9m, did you try computing automorphism group of $\Bbb R$?
the problem I gave you?
 
r9m
@BalarkaSen hehe .. _/_
 
er?
 
r9m
_/\ _
 
3:38 PM
(╯`Д´)╯︵ (_/\ _)
 
@BalarkaSen Help me with anther problem :c
 
Wait a few secs I am answering another question in a forum.
 
Let $P(x)=x^3-x-1$. Show that it has one real root $a$, two complex conjugated roots $b,c$, and that $a^n+b^n+c^n\in\mathbb{Z}$
Shoud be easy for you
 
Look at the discriminant.
And use power-sum formulas.
 
Discriminant ? What's the discriminant of a 3rd degree polynomial ?
 
3:42 PM
heh?
every polynomial has a discriminant!
 
How is it defined
 
google [discriminant].
 
@r9m Shouldn't $$\int_0^1 \int_x^1 \left(\int_x^y u'(t)\,dt\right)^2 \,dx\,dy$$ be $$\int_0^1\int_0^y\left(\int_x^yu'(t) \,\mathrm{d}t\right)^2\,\mathrm{d}x\,\mathrm{d}y$$
 
the usual product definition
 
@BalarkaSen Am I supposed to know that by heart ??
 
3:46 PM
Try deriving it.
 
$3ax^2+2bx+c$
 
What ... are you even doing?
 
I derived it -__-
 
@Hippalectryon Do you even know what is a discriminant?
 
I don't know the general meaning for a polynomial
Only for a matrice
 
3:47 PM
@robjohn Don't scold me. Yesterday I flagged X's comments to Y. Today I flagged Y's comments to X. Again, 10 of them, lol.
 
For a monic polynomial $f(x)$ of deg $n$ with roots $x_k$, it's $$(-1)^{n(n-1)/2}\prod_{i < j} (x_i - x_j) $$ @Hippa
Derive the disc of a cubic from this by expressing it in terms of elementary symmetric polys
 
@Hippalectryon Since that sum runs over the space of very large numbers because of $x\to\infty$, you have a high flexibility, that means you can even try to use Cesaro-Stolz Theorem, the special case $0/0$, some clever inequalities, or even make use of some advanced tools like Euler–Maclaurin formula. I'm confident you can do it on your own (I'm in the meeting).
 
@r9m once that is corrected, the rest is pretty standard (and correct)
 
@Chris'ssis I know the answer, I just don't know how to get to it :) I've already tried , it was on my exam today
With some intermediary questions
 
@Hippa and then prove that if the disc < 0 then two roots appear in complex conjugates
Conclude the first part of your problem. And then use the newton symmetric polynomials and the reccurence relation to finish the next part by induction.
 
r9m
3:51 PM
@robjohn ah right !! it should have been $\,dy \,dx$ (I guess ..)
 
I have given you all the hints you want
Now I need to leave.
Bye.
 
@r9m that would fix it, too.
 
r9m
@robjohn thanks a lot !!!! :D
 
@r9m the question is: why don't I see these questions when they come up :-D
 
@Chris'ssis Nope
 
r9m
3:55 PM
@robjohn ya !!! I had the exact same question in mind yesterday when I saw it =) lol .. I was away for classes and it was asked and answered while I was away =P
 
@r9m I answered that question I mentioned yesterday, and it had set for over 2 years...
 
@Hippalectryon That was just a guess, no calculations. I didn't use pen and paper yet.
 
r9m
@robjohn :O ya !!
 
@robjohn X, Y et al are clearly in cahoots. They form an upvoting ring, lol.
 
bbl (in the meeting)
 
3:57 PM
@WillHunting there is a triangle or perhaps even a quadrangle of them
 
@robjohn Yes, extremely cunning.
 
@WillHunting but they don't trigger the automatic scripts, so the community team doesn't want to intercede. It annoys the mods (at least some of them).
 
@robjohn It is partly my fault. I should not have taught my enemy how to upvote without triggering the script...
 
@WillHunting badbadbadbadbadbad!!!
 
@robjohn That movie is showing in the cinemas now. It is a new movie.
 
4:01 PM
@WillHunting which movie?
 
@robjohn "If I live". I cried throughout the show.
 
r9m
@robjohn the tags for that were (integration) and (inequality) .. I checked the (integral-inequality) tag but didn't check the inequality tag .. that's how I missed it :P
@WillHunting 'how to upvote without triggering the script' ?! :O what is that ?
 
@Hippalectryon $0$?
 
Nope either
@Chris'ssis Oh wait
I just noticed a typo I thought I had fixed
 
r9m
@Chris'ssis you seem to be multitasking between a lot of stuff atm !! :D
 
4:09 PM
We're searching for the limit of $\Gamma(x)$ in $0$, not in $\infty$
 
@Hippalectryon aaaaaaaaaaaaaaaaaaaaaaaaaaaa
 
@Chris'ssis easier ?
 
@Hippalectryon I'll think of this version when I finish the meeting.
@r9m Yeah, meetings ... :-(
 
:D
other people in the meeting: what the hell is he doing, talking on his phone ??
 
r9m
@Chris'ssis you are M.Se chatting during meetings !! Cooler !! ^_^ ;)
 
4:11 PM
@r9m Not between, during :c
 
r9m
@Hippalectryon =P I see !!
 
r9m
4:26 PM
@Hippalectryon what is this about ? :o
 
I have no idea
Nope
Nope either
 
@Hippalectryon I think you're wrong.
 
I have my exam sheet in front of me :/
They give the answer
We have to get to it
 
@Hippalectryon Which is the answer?
 
I didn't manage to do this question though
The answer is $A\gamma$
euler constant
@Chris'ssis Do you want me to post the in-between questions ?
 
4:31 PM
@Hippalectryon No, something is wrong there ...
No ...
 
Hello @TedShifrin yesterday's pics got deleted, lol.
 
I checked my sheet again, I don't think i've made any additional typo
@TedShifrin Wanna try it ?
 
@Hippalectryon How about if I let the variable change $t\mapsto t x$ in $$\displaystyle \gamma_k(x)=xf((k+1)x)-\int_{(k+1)x}^{(k+2)x}f(t)\mathrm{d}t$$? What's next then?
 
r9m
@Anastasiya-Romanova that's a really nice method by Tunk-Fey !! :D
I remember an exercise in Apostol Calculus that asked to evaluate $\sum \frac{1}{n^2}$ in that fashion !! :)
 
Indeed. It made use of Fubini or some such, right?
 
r9m
4:36 PM
yas ! the change of variable was identical .. and I later realized why that sub actually works easier !
 
Good. I didn't forget everything I read about calculus then.
 
@Chris'ssis $\displaystyle\gamma(x)=xf((k+1)x)-\int_{k+1}^{k+2}\frac{f(tx)}x\mathrm{d}(tx)$ ?
I think i forgot something
@Chris'ssis Isn't it $\displaystyle\gamma(x)=xf((k+1)x)-\int_{k+1}^{k+2}f(tx)\cdot x\mathrm{d}(tx)$ ?
 
You write a lot of stuff just for fun ... :-)
(I'm away for some minutes)
@Hippalectryon Now I see why I was wrong. I'll think of a way.
@robjohn if you have time, take a look at the @Hippalectryon's problem, it seems interesting. I'm thinking of a nice way of proving that limit.
 
4:53 PM
0
Q: Limit of the sum of $\gamma_k(x)=xf((k+1)x)-\int_{(k+1)x}^{(k+2)x}f(t)\mathrm{d}t$

HippalectryonLet $f$ be a continuous, decreasing function, with $\displaystyle\lim_{x\rightarrow\infty}=0$. Let $\gamma_k(x)=xf((k+1)x)-\int_{(k+1)x}^{(k+2)x}f(t)\mathrm{d}t,\displaystyle x>0,$. Let $\Gamma(x)=\sum\limits_{k=0}^{\infty}\gamma_k(x)$, suppose that $\displaystyle\lim_{x\rightarrow0} xf(x)=A$. ...

away 30 mins or so
 
@r9m I don't check tags... I just watch the front page unless a link or something brings another to my attention. I fear that I will miss out on some good question that is out of the set of tags I choose. I like problems in a wide variety of topics.
 

« first day (1505 days earlier)      last day (3513 days later) »